Prove that polynomial of degree $4$ with real roots cannot have $pm 1$ as coefficients (IITJEE)

The name of the pictureThe name of the pictureThe name of the pictureClash Royale CLAN TAG#URR8PPP











up vote
10
down vote

favorite
4












So I was going through my 11th class package on Quadratic equations and I saw a question to prove that a polynomial of $4$th degree with all real roots cannot have $pm 1$ as all its coefficients.



I tried proving it using calculus, by showing that at least one consecutive maxima and minima will lie either above or below the x axis, but couldn't solve it using that.



I also tried using Descartes Rule of Signs but couldn't solve it with that too.
Any help?







share|cite|improve this question

















  • 3




    Boring / unenlightening approach: There are, essentially, only $16$ such polynomials. One could just go through each of them, with the help of WolframAlpha, and check. Would only take a minute or two.
    – Arthur
    Jul 16 at 15:25







  • 1




    I like the approaches taken by OP. Note that taking a second derivative might close down the number of cases to consider substantially.
    – hardmath
    Jul 16 at 15:27






  • 1




    To be precise, there are $32$ such polynomials
    – asdf
    Jul 16 at 15:27






  • 4




    @asdf But $f$ and $-f$ have the same roots, so we may WLOG assume the polynomial is monic. That's what I meant by "essentially". In fact, $f(x)$ and $f(-x)$ are closely related too, so $8$ cases left to check.
    – Arthur
    Jul 16 at 15:28











  • True, I thought you aren't narrowing the cases in any way before bashing it out
    – asdf
    Jul 16 at 15:29














up vote
10
down vote

favorite
4












So I was going through my 11th class package on Quadratic equations and I saw a question to prove that a polynomial of $4$th degree with all real roots cannot have $pm 1$ as all its coefficients.



I tried proving it using calculus, by showing that at least one consecutive maxima and minima will lie either above or below the x axis, but couldn't solve it using that.



I also tried using Descartes Rule of Signs but couldn't solve it with that too.
Any help?







share|cite|improve this question

















  • 3




    Boring / unenlightening approach: There are, essentially, only $16$ such polynomials. One could just go through each of them, with the help of WolframAlpha, and check. Would only take a minute or two.
    – Arthur
    Jul 16 at 15:25







  • 1




    I like the approaches taken by OP. Note that taking a second derivative might close down the number of cases to consider substantially.
    – hardmath
    Jul 16 at 15:27






  • 1




    To be precise, there are $32$ such polynomials
    – asdf
    Jul 16 at 15:27






  • 4




    @asdf But $f$ and $-f$ have the same roots, so we may WLOG assume the polynomial is monic. That's what I meant by "essentially". In fact, $f(x)$ and $f(-x)$ are closely related too, so $8$ cases left to check.
    – Arthur
    Jul 16 at 15:28











  • True, I thought you aren't narrowing the cases in any way before bashing it out
    – asdf
    Jul 16 at 15:29












up vote
10
down vote

favorite
4









up vote
10
down vote

favorite
4






4





So I was going through my 11th class package on Quadratic equations and I saw a question to prove that a polynomial of $4$th degree with all real roots cannot have $pm 1$ as all its coefficients.



I tried proving it using calculus, by showing that at least one consecutive maxima and minima will lie either above or below the x axis, but couldn't solve it using that.



I also tried using Descartes Rule of Signs but couldn't solve it with that too.
Any help?







share|cite|improve this question













So I was going through my 11th class package on Quadratic equations and I saw a question to prove that a polynomial of $4$th degree with all real roots cannot have $pm 1$ as all its coefficients.



I tried proving it using calculus, by showing that at least one consecutive maxima and minima will lie either above or below the x axis, but couldn't solve it using that.



I also tried using Descartes Rule of Signs but couldn't solve it with that too.
Any help?









share|cite|improve this question












share|cite|improve this question




share|cite|improve this question








edited Jul 16 at 15:43









StubbornAtom

3,79311134




3,79311134









asked Jul 16 at 15:21









James Adams

535




535







  • 3




    Boring / unenlightening approach: There are, essentially, only $16$ such polynomials. One could just go through each of them, with the help of WolframAlpha, and check. Would only take a minute or two.
    – Arthur
    Jul 16 at 15:25







  • 1




    I like the approaches taken by OP. Note that taking a second derivative might close down the number of cases to consider substantially.
    – hardmath
    Jul 16 at 15:27






  • 1




    To be precise, there are $32$ such polynomials
    – asdf
    Jul 16 at 15:27






  • 4




    @asdf But $f$ and $-f$ have the same roots, so we may WLOG assume the polynomial is monic. That's what I meant by "essentially". In fact, $f(x)$ and $f(-x)$ are closely related too, so $8$ cases left to check.
    – Arthur
    Jul 16 at 15:28











  • True, I thought you aren't narrowing the cases in any way before bashing it out
    – asdf
    Jul 16 at 15:29












  • 3




    Boring / unenlightening approach: There are, essentially, only $16$ such polynomials. One could just go through each of them, with the help of WolframAlpha, and check. Would only take a minute or two.
    – Arthur
    Jul 16 at 15:25







  • 1




    I like the approaches taken by OP. Note that taking a second derivative might close down the number of cases to consider substantially.
    – hardmath
    Jul 16 at 15:27






  • 1




    To be precise, there are $32$ such polynomials
    – asdf
    Jul 16 at 15:27






  • 4




    @asdf But $f$ and $-f$ have the same roots, so we may WLOG assume the polynomial is monic. That's what I meant by "essentially". In fact, $f(x)$ and $f(-x)$ are closely related too, so $8$ cases left to check.
    – Arthur
    Jul 16 at 15:28











  • True, I thought you aren't narrowing the cases in any way before bashing it out
    – asdf
    Jul 16 at 15:29







3




3




Boring / unenlightening approach: There are, essentially, only $16$ such polynomials. One could just go through each of them, with the help of WolframAlpha, and check. Would only take a minute or two.
– Arthur
Jul 16 at 15:25





Boring / unenlightening approach: There are, essentially, only $16$ such polynomials. One could just go through each of them, with the help of WolframAlpha, and check. Would only take a minute or two.
– Arthur
Jul 16 at 15:25





1




1




I like the approaches taken by OP. Note that taking a second derivative might close down the number of cases to consider substantially.
– hardmath
Jul 16 at 15:27




I like the approaches taken by OP. Note that taking a second derivative might close down the number of cases to consider substantially.
– hardmath
Jul 16 at 15:27




1




1




To be precise, there are $32$ such polynomials
– asdf
Jul 16 at 15:27




To be precise, there are $32$ such polynomials
– asdf
Jul 16 at 15:27




4




4




@asdf But $f$ and $-f$ have the same roots, so we may WLOG assume the polynomial is monic. That's what I meant by "essentially". In fact, $f(x)$ and $f(-x)$ are closely related too, so $8$ cases left to check.
– Arthur
Jul 16 at 15:28





@asdf But $f$ and $-f$ have the same roots, so we may WLOG assume the polynomial is monic. That's what I meant by "essentially". In fact, $f(x)$ and $f(-x)$ are closely related too, so $8$ cases left to check.
– Arthur
Jul 16 at 15:28













True, I thought you aren't narrowing the cases in any way before bashing it out
– asdf
Jul 16 at 15:29




True, I thought you aren't narrowing the cases in any way before bashing it out
– asdf
Jul 16 at 15:29










2 Answers
2






active

oldest

votes

















up vote
9
down vote



accepted










Let $f(x)$ be any quartic polynomial with coefficients from $ -1, +1 $. Replacing $f(x)$ by $-f(x)$ if necessary, we can assume $f(x)$ is monic. i.e.



$$f(x) = x^4 + ax^3 + bx^2 + cx + dquadtext with quad a,b,c,d in -1, +1 $$



If $f(x)$ has $4$ real roots $lambda_1,lambda_2,lambda_3,lambda_4$, then by Vieta's formula, we have



$$sum_i=1^4 lambda_i = -a, sum_1le i < jle 4 lambda_ilambda_j = b
quadtext and quadprod_i=1^4 lambda_i = d$$
Notice
$$sum_i=1^4 lambda_i^2 = left(sum_i=1^4lambda_iright)^2 - 2sum_1le i < j le 4lambda_ilambda_j = a^2 - 2b = 1 -2b$$



Since $sum_i=1^4 lambda_i^2 ge 0$, we need $b = -1$. As a result, $$sum_i=1^4 lambda_i^2 = 3$$
By AM $ge$ GM, this leads to



$$frac34 = frac14sum_i=1^4 lambda_i^2 ge left(prod_i=1^4 lambda_i^2right)^1/4 = (d^2)^1/4 = 1$$
This is impossible and hence $f(x)$ cannot has 4 real roots.






share|cite|improve this answer























  • You are not using at any point that the sum of product of triples is $pm1$, so the claim can be generalised?
    – asdf
    Jul 16 at 15:52










  • @asdf yes, the proof doesn't need the value of $c$.
    – achille hui
    Jul 16 at 15:53










  • Nice, $(+1)$ from me
    – asdf
    Jul 16 at 15:53










  • Achille thanks for answering. However I'm an 11th grader and this looks out of my depth(this is in fact from my textbook). So can you explain to me what you have in a more basic manner? Thanks.Again if this cannot be explained to an 11th grader then I would still consider this question as still unanswered since it was designed to be solved by an 11th grader. Thanks a lot again mate.
    – James Adams
    Jul 16 at 15:56







  • 1




    @JamesAdams I don't remember how long I take, it is not that long. Since you are in 11 grade, I stop using anything that need calculus and looks for pure algebraic solution. I first look at tools like Newton inequalities or Maclaurin inequalities for polynomials for hints of an elementary solution (these tools are beyond 11 grade) but that didn't work. I switch to use Vieta's formula and attempt to bound the roots directly. That work and the rest is filling the details.
    – achille hui
    Jul 17 at 16:11

















up vote
5
down vote













It can be assumed WLOG that the leading coefficient is $,+1,$, so $,P(x)=x^4pm x^3pm x^2pm xpm 1,$.



  • Then $,P''(x)=12x^2 pm 6x pm 2,$, and for the quadratic to have real roots it is necessary that the constant term be negative, so $,P(x)=x^4pm x^3 - x^2pm xpm 1,$.


  • $P(x),$ has all the roots real iff $,x^4 Pleft(frac1xright),$ has all real roots. By the same argument as above, the constant term of $,P(x),$ must have opposite sign as the coefficient of $,x^2,$.


This leaves $4$ cases to check $,P(x)=x^4pm x^3 - x^2pm x+1,$.






[ EDIT ]

  • $P(x),$ has all the roots real iff $,Pleft(-xright),$ has all real roots, so it is enough to consider the case where the coefficient of $,x^3,$ is $+1$.

This leaves $2$ cases to check $,P(x)=x^4+ x^3 - x^2pm x+1,$.






share|cite|improve this answer























  • @RossMillikan On the other hand, it is enough to consider one of the cases $,pm x^3,$, as noted in the latest edit.
    – dxiv
    Jul 16 at 16:08











Your Answer




StackExchange.ifUsing("editor", function ()
return StackExchange.using("mathjaxEditing", function ()
StackExchange.MarkdownEditor.creationCallbacks.add(function (editor, postfix)
StackExchange.mathjaxEditing.prepareWmdForMathJax(editor, postfix, [["$", "$"], ["\\(","\\)"]]);
);
);
, "mathjax-editing");

StackExchange.ready(function()
var channelOptions =
tags: "".split(" "),
id: "69"
;
initTagRenderer("".split(" "), "".split(" "), channelOptions);

StackExchange.using("externalEditor", function()
// Have to fire editor after snippets, if snippets enabled
if (StackExchange.settings.snippets.snippetsEnabled)
StackExchange.using("snippets", function()
createEditor();
);

else
createEditor();

);

function createEditor()
StackExchange.prepareEditor(
heartbeatType: 'answer',
convertImagesToLinks: true,
noModals: false,
showLowRepImageUploadWarning: true,
reputationToPostImages: 10,
bindNavPrevention: true,
postfix: "",
noCode: true, onDemand: true,
discardSelector: ".discard-answer"
,immediatelyShowMarkdownHelp:true
);



);








 

draft saved


draft discarded


















StackExchange.ready(
function ()
StackExchange.openid.initPostLogin('.new-post-login', 'https%3a%2f%2fmath.stackexchange.com%2fquestions%2f2853502%2fprove-that-polynomial-of-degree-4-with-real-roots-cannot-have-pm-1-as-coeff%23new-answer', 'question_page');

);

Post as a guest






























2 Answers
2






active

oldest

votes








2 Answers
2






active

oldest

votes









active

oldest

votes






active

oldest

votes








up vote
9
down vote



accepted










Let $f(x)$ be any quartic polynomial with coefficients from $ -1, +1 $. Replacing $f(x)$ by $-f(x)$ if necessary, we can assume $f(x)$ is monic. i.e.



$$f(x) = x^4 + ax^3 + bx^2 + cx + dquadtext with quad a,b,c,d in -1, +1 $$



If $f(x)$ has $4$ real roots $lambda_1,lambda_2,lambda_3,lambda_4$, then by Vieta's formula, we have



$$sum_i=1^4 lambda_i = -a, sum_1le i < jle 4 lambda_ilambda_j = b
quadtext and quadprod_i=1^4 lambda_i = d$$
Notice
$$sum_i=1^4 lambda_i^2 = left(sum_i=1^4lambda_iright)^2 - 2sum_1le i < j le 4lambda_ilambda_j = a^2 - 2b = 1 -2b$$



Since $sum_i=1^4 lambda_i^2 ge 0$, we need $b = -1$. As a result, $$sum_i=1^4 lambda_i^2 = 3$$
By AM $ge$ GM, this leads to



$$frac34 = frac14sum_i=1^4 lambda_i^2 ge left(prod_i=1^4 lambda_i^2right)^1/4 = (d^2)^1/4 = 1$$
This is impossible and hence $f(x)$ cannot has 4 real roots.






share|cite|improve this answer























  • You are not using at any point that the sum of product of triples is $pm1$, so the claim can be generalised?
    – asdf
    Jul 16 at 15:52










  • @asdf yes, the proof doesn't need the value of $c$.
    – achille hui
    Jul 16 at 15:53










  • Nice, $(+1)$ from me
    – asdf
    Jul 16 at 15:53










  • Achille thanks for answering. However I'm an 11th grader and this looks out of my depth(this is in fact from my textbook). So can you explain to me what you have in a more basic manner? Thanks.Again if this cannot be explained to an 11th grader then I would still consider this question as still unanswered since it was designed to be solved by an 11th grader. Thanks a lot again mate.
    – James Adams
    Jul 16 at 15:56







  • 1




    @JamesAdams I don't remember how long I take, it is not that long. Since you are in 11 grade, I stop using anything that need calculus and looks for pure algebraic solution. I first look at tools like Newton inequalities or Maclaurin inequalities for polynomials for hints of an elementary solution (these tools are beyond 11 grade) but that didn't work. I switch to use Vieta's formula and attempt to bound the roots directly. That work and the rest is filling the details.
    – achille hui
    Jul 17 at 16:11














up vote
9
down vote



accepted










Let $f(x)$ be any quartic polynomial with coefficients from $ -1, +1 $. Replacing $f(x)$ by $-f(x)$ if necessary, we can assume $f(x)$ is monic. i.e.



$$f(x) = x^4 + ax^3 + bx^2 + cx + dquadtext with quad a,b,c,d in -1, +1 $$



If $f(x)$ has $4$ real roots $lambda_1,lambda_2,lambda_3,lambda_4$, then by Vieta's formula, we have



$$sum_i=1^4 lambda_i = -a, sum_1le i < jle 4 lambda_ilambda_j = b
quadtext and quadprod_i=1^4 lambda_i = d$$
Notice
$$sum_i=1^4 lambda_i^2 = left(sum_i=1^4lambda_iright)^2 - 2sum_1le i < j le 4lambda_ilambda_j = a^2 - 2b = 1 -2b$$



Since $sum_i=1^4 lambda_i^2 ge 0$, we need $b = -1$. As a result, $$sum_i=1^4 lambda_i^2 = 3$$
By AM $ge$ GM, this leads to



$$frac34 = frac14sum_i=1^4 lambda_i^2 ge left(prod_i=1^4 lambda_i^2right)^1/4 = (d^2)^1/4 = 1$$
This is impossible and hence $f(x)$ cannot has 4 real roots.






share|cite|improve this answer























  • You are not using at any point that the sum of product of triples is $pm1$, so the claim can be generalised?
    – asdf
    Jul 16 at 15:52










  • @asdf yes, the proof doesn't need the value of $c$.
    – achille hui
    Jul 16 at 15:53










  • Nice, $(+1)$ from me
    – asdf
    Jul 16 at 15:53










  • Achille thanks for answering. However I'm an 11th grader and this looks out of my depth(this is in fact from my textbook). So can you explain to me what you have in a more basic manner? Thanks.Again if this cannot be explained to an 11th grader then I would still consider this question as still unanswered since it was designed to be solved by an 11th grader. Thanks a lot again mate.
    – James Adams
    Jul 16 at 15:56







  • 1




    @JamesAdams I don't remember how long I take, it is not that long. Since you are in 11 grade, I stop using anything that need calculus and looks for pure algebraic solution. I first look at tools like Newton inequalities or Maclaurin inequalities for polynomials for hints of an elementary solution (these tools are beyond 11 grade) but that didn't work. I switch to use Vieta's formula and attempt to bound the roots directly. That work and the rest is filling the details.
    – achille hui
    Jul 17 at 16:11












up vote
9
down vote



accepted







up vote
9
down vote



accepted






Let $f(x)$ be any quartic polynomial with coefficients from $ -1, +1 $. Replacing $f(x)$ by $-f(x)$ if necessary, we can assume $f(x)$ is monic. i.e.



$$f(x) = x^4 + ax^3 + bx^2 + cx + dquadtext with quad a,b,c,d in -1, +1 $$



If $f(x)$ has $4$ real roots $lambda_1,lambda_2,lambda_3,lambda_4$, then by Vieta's formula, we have



$$sum_i=1^4 lambda_i = -a, sum_1le i < jle 4 lambda_ilambda_j = b
quadtext and quadprod_i=1^4 lambda_i = d$$
Notice
$$sum_i=1^4 lambda_i^2 = left(sum_i=1^4lambda_iright)^2 - 2sum_1le i < j le 4lambda_ilambda_j = a^2 - 2b = 1 -2b$$



Since $sum_i=1^4 lambda_i^2 ge 0$, we need $b = -1$. As a result, $$sum_i=1^4 lambda_i^2 = 3$$
By AM $ge$ GM, this leads to



$$frac34 = frac14sum_i=1^4 lambda_i^2 ge left(prod_i=1^4 lambda_i^2right)^1/4 = (d^2)^1/4 = 1$$
This is impossible and hence $f(x)$ cannot has 4 real roots.






share|cite|improve this answer















Let $f(x)$ be any quartic polynomial with coefficients from $ -1, +1 $. Replacing $f(x)$ by $-f(x)$ if necessary, we can assume $f(x)$ is monic. i.e.



$$f(x) = x^4 + ax^3 + bx^2 + cx + dquadtext with quad a,b,c,d in -1, +1 $$



If $f(x)$ has $4$ real roots $lambda_1,lambda_2,lambda_3,lambda_4$, then by Vieta's formula, we have



$$sum_i=1^4 lambda_i = -a, sum_1le i < jle 4 lambda_ilambda_j = b
quadtext and quadprod_i=1^4 lambda_i = d$$
Notice
$$sum_i=1^4 lambda_i^2 = left(sum_i=1^4lambda_iright)^2 - 2sum_1le i < j le 4lambda_ilambda_j = a^2 - 2b = 1 -2b$$



Since $sum_i=1^4 lambda_i^2 ge 0$, we need $b = -1$. As a result, $$sum_i=1^4 lambda_i^2 = 3$$
By AM $ge$ GM, this leads to



$$frac34 = frac14sum_i=1^4 lambda_i^2 ge left(prod_i=1^4 lambda_i^2right)^1/4 = (d^2)^1/4 = 1$$
This is impossible and hence $f(x)$ cannot has 4 real roots.







share|cite|improve this answer















share|cite|improve this answer



share|cite|improve this answer








edited Jul 16 at 15:59


























answered Jul 16 at 15:48









achille hui

91k5127246




91k5127246











  • You are not using at any point that the sum of product of triples is $pm1$, so the claim can be generalised?
    – asdf
    Jul 16 at 15:52










  • @asdf yes, the proof doesn't need the value of $c$.
    – achille hui
    Jul 16 at 15:53










  • Nice, $(+1)$ from me
    – asdf
    Jul 16 at 15:53










  • Achille thanks for answering. However I'm an 11th grader and this looks out of my depth(this is in fact from my textbook). So can you explain to me what you have in a more basic manner? Thanks.Again if this cannot be explained to an 11th grader then I would still consider this question as still unanswered since it was designed to be solved by an 11th grader. Thanks a lot again mate.
    – James Adams
    Jul 16 at 15:56







  • 1




    @JamesAdams I don't remember how long I take, it is not that long. Since you are in 11 grade, I stop using anything that need calculus and looks for pure algebraic solution. I first look at tools like Newton inequalities or Maclaurin inequalities for polynomials for hints of an elementary solution (these tools are beyond 11 grade) but that didn't work. I switch to use Vieta's formula and attempt to bound the roots directly. That work and the rest is filling the details.
    – achille hui
    Jul 17 at 16:11
















  • You are not using at any point that the sum of product of triples is $pm1$, so the claim can be generalised?
    – asdf
    Jul 16 at 15:52










  • @asdf yes, the proof doesn't need the value of $c$.
    – achille hui
    Jul 16 at 15:53










  • Nice, $(+1)$ from me
    – asdf
    Jul 16 at 15:53










  • Achille thanks for answering. However I'm an 11th grader and this looks out of my depth(this is in fact from my textbook). So can you explain to me what you have in a more basic manner? Thanks.Again if this cannot be explained to an 11th grader then I would still consider this question as still unanswered since it was designed to be solved by an 11th grader. Thanks a lot again mate.
    – James Adams
    Jul 16 at 15:56







  • 1




    @JamesAdams I don't remember how long I take, it is not that long. Since you are in 11 grade, I stop using anything that need calculus and looks for pure algebraic solution. I first look at tools like Newton inequalities or Maclaurin inequalities for polynomials for hints of an elementary solution (these tools are beyond 11 grade) but that didn't work. I switch to use Vieta's formula and attempt to bound the roots directly. That work and the rest is filling the details.
    – achille hui
    Jul 17 at 16:11















You are not using at any point that the sum of product of triples is $pm1$, so the claim can be generalised?
– asdf
Jul 16 at 15:52




You are not using at any point that the sum of product of triples is $pm1$, so the claim can be generalised?
– asdf
Jul 16 at 15:52












@asdf yes, the proof doesn't need the value of $c$.
– achille hui
Jul 16 at 15:53




@asdf yes, the proof doesn't need the value of $c$.
– achille hui
Jul 16 at 15:53












Nice, $(+1)$ from me
– asdf
Jul 16 at 15:53




Nice, $(+1)$ from me
– asdf
Jul 16 at 15:53












Achille thanks for answering. However I'm an 11th grader and this looks out of my depth(this is in fact from my textbook). So can you explain to me what you have in a more basic manner? Thanks.Again if this cannot be explained to an 11th grader then I would still consider this question as still unanswered since it was designed to be solved by an 11th grader. Thanks a lot again mate.
– James Adams
Jul 16 at 15:56





Achille thanks for answering. However I'm an 11th grader and this looks out of my depth(this is in fact from my textbook). So can you explain to me what you have in a more basic manner? Thanks.Again if this cannot be explained to an 11th grader then I would still consider this question as still unanswered since it was designed to be solved by an 11th grader. Thanks a lot again mate.
– James Adams
Jul 16 at 15:56





1




1




@JamesAdams I don't remember how long I take, it is not that long. Since you are in 11 grade, I stop using anything that need calculus and looks for pure algebraic solution. I first look at tools like Newton inequalities or Maclaurin inequalities for polynomials for hints of an elementary solution (these tools are beyond 11 grade) but that didn't work. I switch to use Vieta's formula and attempt to bound the roots directly. That work and the rest is filling the details.
– achille hui
Jul 17 at 16:11




@JamesAdams I don't remember how long I take, it is not that long. Since you are in 11 grade, I stop using anything that need calculus and looks for pure algebraic solution. I first look at tools like Newton inequalities or Maclaurin inequalities for polynomials for hints of an elementary solution (these tools are beyond 11 grade) but that didn't work. I switch to use Vieta's formula and attempt to bound the roots directly. That work and the rest is filling the details.
– achille hui
Jul 17 at 16:11










up vote
5
down vote













It can be assumed WLOG that the leading coefficient is $,+1,$, so $,P(x)=x^4pm x^3pm x^2pm xpm 1,$.



  • Then $,P''(x)=12x^2 pm 6x pm 2,$, and for the quadratic to have real roots it is necessary that the constant term be negative, so $,P(x)=x^4pm x^3 - x^2pm xpm 1,$.


  • $P(x),$ has all the roots real iff $,x^4 Pleft(frac1xright),$ has all real roots. By the same argument as above, the constant term of $,P(x),$ must have opposite sign as the coefficient of $,x^2,$.


This leaves $4$ cases to check $,P(x)=x^4pm x^3 - x^2pm x+1,$.






[ EDIT ]

  • $P(x),$ has all the roots real iff $,Pleft(-xright),$ has all real roots, so it is enough to consider the case where the coefficient of $,x^3,$ is $+1$.

This leaves $2$ cases to check $,P(x)=x^4+ x^3 - x^2pm x+1,$.






share|cite|improve this answer























  • @RossMillikan On the other hand, it is enough to consider one of the cases $,pm x^3,$, as noted in the latest edit.
    – dxiv
    Jul 16 at 16:08















up vote
5
down vote













It can be assumed WLOG that the leading coefficient is $,+1,$, so $,P(x)=x^4pm x^3pm x^2pm xpm 1,$.



  • Then $,P''(x)=12x^2 pm 6x pm 2,$, and for the quadratic to have real roots it is necessary that the constant term be negative, so $,P(x)=x^4pm x^3 - x^2pm xpm 1,$.


  • $P(x),$ has all the roots real iff $,x^4 Pleft(frac1xright),$ has all real roots. By the same argument as above, the constant term of $,P(x),$ must have opposite sign as the coefficient of $,x^2,$.


This leaves $4$ cases to check $,P(x)=x^4pm x^3 - x^2pm x+1,$.






[ EDIT ]

  • $P(x),$ has all the roots real iff $,Pleft(-xright),$ has all real roots, so it is enough to consider the case where the coefficient of $,x^3,$ is $+1$.

This leaves $2$ cases to check $,P(x)=x^4+ x^3 - x^2pm x+1,$.






share|cite|improve this answer























  • @RossMillikan On the other hand, it is enough to consider one of the cases $,pm x^3,$, as noted in the latest edit.
    – dxiv
    Jul 16 at 16:08













up vote
5
down vote










up vote
5
down vote









It can be assumed WLOG that the leading coefficient is $,+1,$, so $,P(x)=x^4pm x^3pm x^2pm xpm 1,$.



  • Then $,P''(x)=12x^2 pm 6x pm 2,$, and for the quadratic to have real roots it is necessary that the constant term be negative, so $,P(x)=x^4pm x^3 - x^2pm xpm 1,$.


  • $P(x),$ has all the roots real iff $,x^4 Pleft(frac1xright),$ has all real roots. By the same argument as above, the constant term of $,P(x),$ must have opposite sign as the coefficient of $,x^2,$.


This leaves $4$ cases to check $,P(x)=x^4pm x^3 - x^2pm x+1,$.






[ EDIT ]

  • $P(x),$ has all the roots real iff $,Pleft(-xright),$ has all real roots, so it is enough to consider the case where the coefficient of $,x^3,$ is $+1$.

This leaves $2$ cases to check $,P(x)=x^4+ x^3 - x^2pm x+1,$.






share|cite|improve this answer















It can be assumed WLOG that the leading coefficient is $,+1,$, so $,P(x)=x^4pm x^3pm x^2pm xpm 1,$.



  • Then $,P''(x)=12x^2 pm 6x pm 2,$, and for the quadratic to have real roots it is necessary that the constant term be negative, so $,P(x)=x^4pm x^3 - x^2pm xpm 1,$.


  • $P(x),$ has all the roots real iff $,x^4 Pleft(frac1xright),$ has all real roots. By the same argument as above, the constant term of $,P(x),$ must have opposite sign as the coefficient of $,x^2,$.


This leaves $4$ cases to check $,P(x)=x^4pm x^3 - x^2pm x+1,$.






[ EDIT ]

  • $P(x),$ has all the roots real iff $,Pleft(-xright),$ has all real roots, so it is enough to consider the case where the coefficient of $,x^3,$ is $+1$.

This leaves $2$ cases to check $,P(x)=x^4+ x^3 - x^2pm x+1,$.







share|cite|improve this answer















share|cite|improve this answer



share|cite|improve this answer








edited Jul 16 at 16:06


























answered Jul 16 at 15:52









dxiv

54.3k64797




54.3k64797











  • @RossMillikan On the other hand, it is enough to consider one of the cases $,pm x^3,$, as noted in the latest edit.
    – dxiv
    Jul 16 at 16:08

















  • @RossMillikan On the other hand, it is enough to consider one of the cases $,pm x^3,$, as noted in the latest edit.
    – dxiv
    Jul 16 at 16:08
















@RossMillikan On the other hand, it is enough to consider one of the cases $,pm x^3,$, as noted in the latest edit.
– dxiv
Jul 16 at 16:08





@RossMillikan On the other hand, it is enough to consider one of the cases $,pm x^3,$, as noted in the latest edit.
– dxiv
Jul 16 at 16:08













 

draft saved


draft discarded


























 


draft saved


draft discarded














StackExchange.ready(
function ()
StackExchange.openid.initPostLogin('.new-post-login', 'https%3a%2f%2fmath.stackexchange.com%2fquestions%2f2853502%2fprove-that-polynomial-of-degree-4-with-real-roots-cannot-have-pm-1-as-coeff%23new-answer', 'question_page');

);

Post as a guest













































































Comments

Popular posts from this blog

Color the edges and diagonals of a regular polygon

Relationship between determinant of matrix and determinant of adjoint?

What is the equation of a 3D cone with generalised tilt?